Cálculo

Qual é o limite de (1+ (4 / x)) ^ x quando x se aproxima do infinito?

Qual é o limite de (1+ (4 / x)) ^ x quando x se aproxima do infinito?

E ^ 4 Observe a definição binomial para o número de Euler: e = lim_ (x-> oo) (1 + 1 / x) ^ x- = lim_ (x-> 0) (1 + x) ^ (1 / x) Aqui Eu vou usar a definição x-> oo. Nessa fórmula, seja y = nx Então 1 / x = n / y, e x = y / n O número de Euler é então expresso de uma forma mais geral: e = lim_ (y-> oo) (1 + n / y) ^ (y / n) Em outras palavras, e ^ n = lim_ (y-> oo) (1 + n / y) ^ y Como y também é uma variável, podemos substituir x em lugar de y: e ^ n = lim_ (x-> oo) (1 + n / x) ^ x Portanto, quando n = 4, lim_ (x-> oo) (1 + 4 / x) ^ x = e Consulte Mais informação »

Qual é o limite de ((1 / x) - ((1) / (e ^ (x) -1)) quando x se aproxima de 0 ^ +?

Qual é o limite de ((1 / x) - ((1) / (e ^ (x) -1)) quando x se aproxima de 0 ^ +?

Lim_ (x rarr 0 ^ +) 1 / x- (1) / (e ^ x-1) = 1/2 Seja: f (x) = 1 / x- (1) / (e ^ x-1) " "= ((e ^ x-1) - (x)) / (x (e ^ x-1))" "= (e ^ x-1 - x) / (xe ^ xx) Então nós procuramos: L = lim_ (x rarr 0 ^ +) f (x) = lim_ (x rarr 0 ^ +) (e ^ x-1 - x) / (xe ^ xx) Como isso é de uma forma indeterminada 0/0 podemos aplicar a regra do L'Hôpital. L = lim_ (x rarr 0 ^ +) (d / dx (e ^ x-1 - x)) / (d / dx (xe ^ xx)) = lim_ (x rarr 0 ^ +) (e ^ x -1) / (xe ^ x + e ^ x - 1) Novamente, isto é de uma forma indeterminada 0/0 podemos aplicar aplicar a regra de L'Hôpital novamente: L = li Consulte Mais informação »

Qual é o limite de ((1) / (x)) - ((1) / (e ^ (x) -1)) quando x se aproxima do infinito?

Qual é o limite de ((1) / (x)) - ((1) / (e ^ (x) -1)) quando x se aproxima do infinito?

Se dois limites adicionados individualmente se aproximarem de 0, a coisa toda se aproxima de 0. Use a propriedade que limita a distribuição sobre adição e subtração. => lim_ (x-> oo) 1 / x - lim_ (x -> oo) 1 / (e ^ x - 1) O primeiro limite é trivial; 1 / "grande" ~~ 0. O segundo lhe pede para saber que e ^ x aumenta quando x aumenta. Por isso, como x-> oo, e ^ x -> oo. => cor (azul) (lim_ (x-> oo) 1 / x - 1 / (e ^ x - 1)) = 1 / oo - 1 / (oo - cancelar (1) ^ "pequeno") = 0 - 0 = cor (azul) (0) Consulte Mais informação »

O que é lim_ (xto0 ^ +) ((1 / x) - ((1) / (e ^ (x) -1)))?

O que é lim_ (xto0 ^ +) ((1 / x) - ((1) / (e ^ (x) -1)))?

Lim_ (x-> 0 ^ +) (1 / x-1 / (e ^ x-1)) = 1/2 Soma os dois termos: 1 / x-1 / (e ^ x-1) = (xe ^ x + 1) / (x (e ^ x-1)) O limite está agora na forma indeterminada 0/0, de modo que agora podemos aplicar a regra de l'Hospital: lim_ (x-> 0 ^ +) (1 / x- 1 / (e ^ x-1)) = lim_ (x-> 0 ^ +) (d / dx (e ^ x + 1-x)) / (d / dx x (e ^ x-1)) lim_ ( x-> 0 ^ +) (1 / x-1 / (e ^ x-1)) = lim_ (x-> 0 ^ +) (e ^ x-1) / (e ^ x-1 + xe ^ x ) e como isso é até na forma 0/0 uma segunda vez: lim_ (x-> 0 ^ +) (1 / x-1 / (e ^ x-1)) = lim_ (x-> 0 ^ +) (d / dx (e ^ x-1)) / (d / dx (e ^ x-1 + xe ^ x)) lim_ (x-> 0 ^ Consulte Mais informação »

Qual é o limite de 7/4 (x-1) ^ 2 quando x se aproxima de 1?

Qual é o limite de 7/4 (x-1) ^ 2 quando x se aproxima de 1?

Lim_ (x-> 1) 7/4 (x-1) ^ 2 = 0 Sabemos que f (x) = 7/4 (x-1) ^ 2 = 0 é contínuo sobre o seu domínio. Então lim_ (x-> c) f (x) = f (c) para todo x no domínio de f. Assim lim_ (x-> 1) 7/4 (x-1) ^ 2 = 7/4 (1-1) ^ 2 = 0 Consulte Mais informação »

Qual é o limite de 7 / (4 (x-1) ^ 2) quando x se aproxima de 1?

Qual é o limite de 7 / (4 (x-1) ^ 2) quando x se aproxima de 1?

Veja abaixo Primeiro, reescreva isso como lim_ (x-> 1) 7 / (4 (x-1) ^ 2 agora fator (x-1) ^ 2 = (x-1) (x-1) = x ^ 2- 2x + 1 frac {7} {4x ^ 2-2x + 1} agora substitua x -> 1 frac {7} {4 (1) ^ 2 -2 (1) +1 7/3 portanto lim_ (x > 1) 7 / (4 (x-1) ^ 2) = 7/6 Consulte Mais informação »

Qual é o lim_ (xrarr1 ^ +) x ^ (1 / (1-x)) quando x se aproxima de 1 do lado direito?

Qual é o lim_ (xrarr1 ^ +) x ^ (1 / (1-x)) quando x se aproxima de 1 do lado direito?

1 / ex ^ (1 / (1-x)): graph {x ^ (1 / (1-x)) [-2,064, 4,095, -1,338, 1,74]} Bem, isso seria muito mais fácil se simplesmente o ln de ambos os lados. Como x ^ (1 / (1-x)) é contínuo no intervalo aberto à direita de 1, podemos dizer que: ln [lim_ (x-> 1 ^ (+)) x ^ (1 / (1- x)]] = lim_ (x-> 1 ^ (+)) ln (x ^ (1 / (1-x))) = lim_ (x-> 1 ^ (+)) ln x / (1-x) Como ln (1) = 0 e (1 - 1) = 0, esta é a forma 0/0 e a regra de L'Hopital se aplica: = lim_ (x-> 1 ^ (+)) (1 "/" x) / (- 1) E, claro, 1 / x é contínuo de cada lado de x = 1. => ln [lim_ (x-> 1 ^ (+)) x ^ (1 / (1 Consulte Mais informação »

Qual é a aproximação linear de g (x) = sqrt (1 + x) ^ (1/5) em a = 0?

Qual é a aproximação linear de g (x) = sqrt (1 + x) ^ (1/5) em a = 0?

(Eu suponho que você quer dizer x = 0) A função, usando as propriedades de energia, torna-se: y = ((1 + x) ^ (1/2)) ^ (1/5) = (1 + x) ^ (( 1/2) (1/5)) = (1 + x) ^ (1/10) Para fazer uma aproximação linear desta função é útil lembrar a série MacLaurin, que é o polinômio de Taylor centrado em zero. Esta série, interrompida para a segunda potência, é: (1 + x) ^ alpha = 1 + alfa / (1!) X + (alfa (alfa-1)) / (2!) X ^ 2 ... então o linear A aproximação desta função é: g (x) = 1 + 1 / 10x Consulte Mais informação »

Qual é a linha de simetria do gráfico de y = 1 / (x-1)?

Qual é a linha de simetria do gráfico de y = 1 / (x-1)?

O gráfico é uma hipérbole, portanto há duas linhas de simetria: y = x-1 e y = -x + 1 O gráfico de y = 1 / (x-1) é uma hipérbole. As hipérboles têm duas linhas de simetria. ambas as linhas de simetria passam pelo centro da hipérbole. Um passa pelos vértices (e pelos focos) e o outro é perpendicular ao primeiro. O gráfico de y = 1 / (x-1) é uma tradução do gráfico de y = 1 / x. y = 1 / x tem centro (0,0) e dois de simetria: y = x e y = -x Para y = 1 / (x-1) substituímos x por x-1 (e não substituímos y Isso traduz o centro para Consulte Mais informação »

Como você diferencia f (x) = (x ^ 3-2x + 3) ^ (3/2) usando a regra da cadeia?

Como você diferencia f (x) = (x ^ 3-2x + 3) ^ (3/2) usando a regra da cadeia?

3/2 * (sqrt (x ^ 3 - 2x + 3)) * (3x ^ 2 - 2) A regra da cadeia: d / dx f (g (x)) = f '(g (x)) * g' (x) A regra de poder: d / dx x ^ n = n * x ^ (n-1) Aplicando estas regras: 1 A função interna, g (x) é x ^ 3-2x + 3, a função externa f (x) é g (x) ^ (3/2) 2 Pegue a derivada da função externa usando a regra de potência d / dx (g (x)) ^ (3/2) = 3/2 * g (x) ^ (3/2 - 2/2) = 3/2 * g (x) ^ (1/2) = 3/2 * sqrt (g (x)) f '(g (x)) = 3/2 * sqrt (x ^ 3 - 2x + 3) 3 Pegue a derivada da função interna d / dx g (x) = 3x ^ 2 -2 g '(x) = 3x ^ 2 -2 4 Multiplique f' (g Consulte Mais informação »

Como você integra int x ^ 2 e ^ (- x) dx usando integração por partes?

Como você integra int x ^ 2 e ^ (- x) dx usando integração por partes?

Intx ^ 2e ^ (- x) dx = -e ^ (- x) (x ^ 2 + 2x + 2) + C Integração por partes diz que: intv (du) / (dx) = uv-intu (dv) / (dx) u = x ^ 2; (du) / (dx) = 2x (dv) / (dx) = e ^ (- x); v = -e ^ (- x) intx ^ 2e ^ (- x) dx = -x ^ 2e ^ (- x) -int-2xe ^ (- 2x) dx Agora fazemos isso: int-2xe ^ (- 2x) dx u = 2x; (du) / (dx) = 2 (dv ) / (dx) = - e ^ (- x); v = e ^ (- x) int-2xe ^ (- x) dx = 2xe ^ (- x) -int2e ^ (- x) dx = 2xe ^ ( -x) + 2e ^ (- x) intx ^ 2e ^ (- x) dx = -x ^ 2e ^ (- x) - (2xe ^ (- x) + 2e ^ (- x)) = - x ^ 2e ^ (- x) -2xe ^ (- x) -2e ^ (- x) + C = -e ^ (- x) (x ^ 2 + 2x + 2) + C Consulte Mais informação »

Qual é a equação da linha normal para f (x) = sec4x-cot2x em x = pi / 3?

Qual é a equação da linha normal para f (x) = sec4x-cot2x em x = pi / 3?

"Normal" => y = - (3x) / (8-24sqrt3) + (152sqrt3-120 + 3pi) / (24-72sqrt2) => y ~ ~ 0,089x-1,52 O normal é a linha perpendicular à tangente. f (x) = s (4x) -cot (2x) f '(x) = 4 seg (4x) tan (3x) + 2 seg 2 (2x) f' (pi / 3) = 4 seg ((4pi) / 3 ) tan ((4pi) / 3) + 2csc ^ 2 ((2pi) / 3) = (8-24sqrt3) / 3 Para normal, m = -1 / (f '(pi / 3)) = - 3 / ( 8-24sqrt3) f (pi / 3) = seg ((4pi) / 3) --cot ((2pi) / 3) = (sqrt3-6) / 3 (sqrt3-6) / 3 = -3 / (8- 24sqrt3) (pi / 3) + cc = (sqrt3-6) / 3 + pi / (8-24sqrt3) = (152sqrt3-120 + 3pi) / (24-72sqrt2) "Normal": y = - (3x) / (8-24sqrt3) + (152 Consulte Mais informação »

Qual é a taxa máxima de mudança de f (x, y) = y ^ 2 / x no ponto 2,4?

Qual é a taxa máxima de mudança de f (x, y) = y ^ 2 / x no ponto 2,4?

Eu acho que você está perguntando sobre a derivada direcional aqui, e a taxa máxima de mudança que é o gradiente, levando ao vetor normal vec n. Assim, para escalar f (x, y) = y ^ 2 / x, podemos dizer que: nabla vec f = langle - y ^ 2 / x ^ 2, (2y) / x rangle = vec n E: vec n _ {( 2,4)} = nabla f _ {(2,4)} = langle -4, 4 rangle Assim, podemos concluir que: abs (vec n _ {(2,4)}) = abs (langle -4, 4 rangle) = 2 sqrt2 Consulte Mais informação »

Qual é o valor máximo de (3-cosx) / (1 + cosx) para 0 <x <(2pi)?

Qual é o valor máximo de (3-cosx) / (1 + cosx) para 0 <x <(2pi)?

X_ {max} = + infty x_ {min} = 0 A função tem uma assíntota vertical em x = pi e seu máximo é quando o denominador tem o menor valor apenas para x = + pi, em vez disso é mínimo quando o denominador é o maior ou sejapara x = 0 e x = 2pi A mesma conclusão poderia ter sido deduzida derivando a função e estudando o sinal da primeira derivada! Consulte Mais informação »

Qual é o significado da forma indeterminada? E se possível, uma lista de todas as formas indeterminadas?

Qual é o significado da forma indeterminada? E se possível, uma lista de todas as formas indeterminadas?

Primeiro de tudo, não há números indeterminados. Há números e há descrições que parecem descrever um número, mas não. "O número x que faz x + 3 = x-5" é tal descrição. Como é "O número 0/0". É melhor evitar dizer (e pensar) que "0/0 é um número indeterminado". . No contexto dos limites: Ao avaliar um limite de uma função "construída" por alguma combinação algébrica de funções, usamos as propriedades dos limites. Aqui estão alguns dos. Observe a c Consulte Mais informação »

Qual é o valor mínimo de f (x) = 3x ^ 2-6x + 12?

Qual é o valor mínimo de f (x) = 3x ^ 2-6x + 12?

9 Os pontos mínimo e máximo relativos podem ser encontrados configurando a derivada para zero. Nesse caso, f '(x) = 0 iff6x-6 = 0 iff x = 1 O valor da função correspondente em 1 é f (1) = 9. Portanto, o ponto (1,9) é um ponto extremo relativo. Como a segunda derivada é positiva quando x = 1, f '' (1) = 6> 0, isso implica que x = 1 é um mínimo relativo. Como a função f é um polinômio de 2º grau, seu gráfico é uma parábola e, portanto, f (x) = 9 também é o mínimo absoluto da função sobre (-oo, oo). O g Consulte Mais informação »

Qual é o valor mínimo de g (x) = (x-1) / (x ^ 2 + 4)? no intervalo [-2,2]?

Qual é o valor mínimo de g (x) = (x-1) / (x ^ 2 + 4)? no intervalo [-2,2]?

O valor mínimo está em x = 1-sqrt 5 approx "-" 1,236; g (1 - sqrt 5) = - (1+ sqrt 5) / (8) aprox "-" 0,405. Em um intervalo fechado, os possíveis locais para um mínimo serão: um mínimo local dentro do intervalo ou os pontos finais do intervalo. Portanto, calculamos e comparamos valores para g (x) em qualquer x em ["-2", 2] que faz g '(x) = 0, assim como em x = "- 2" e x = 2. Primeiro: o que é g '(x)? Usando a regra do quociente, obtemos: g '(x) = ((1) (x ^ 2 + 4) - (x-1) (2x)) / (x ^ 2 + 4) ^ 2 cor (branco) ( g '(x)) = (x ^ 2 + 4-2x Consulte Mais informação »

Qual é o valor mínimo de g (x) = x ^ 2-2x - 11 / x? no intervalo [1,7]?

Qual é o valor mínimo de g (x) = x ^ 2-2x - 11 / x? no intervalo [1,7]?

A função está aumentando continuamente no intervalo [1,7] seu valor mínimo está em x = 1. É óbvio que x ^ 2-2x-11 / x não está definido em x = 0, no entanto, é definido no intervalo [1,7]. Agora derivada de x ^ 2-2x-11 / x é 2x-2 - (- 11 / x ^ 2) ou 2x-2 + 11 / x ^ 2 e é positiva ao longo de [1,7] Assim, a função é continuamente aumentando no intervalo [1,7] e como tal valor mínimo de x ^ 2-2x-11 / x no intervalo [1,7] está em x = 1. gráfico {x ^ 2-2x-11 / x [-40, 40, -20, 20]} Consulte Mais informação »

Qual é o valor mínimo de g (x) = x / csc (pi * x) no intervalo [0,1]?

Qual é o valor mínimo de g (x) = x / csc (pi * x) no intervalo [0,1]?

Existe um valor mínimo de 0 localizado em x = 0 e x = 1. Primeiro, podemos escrever imediatamente esta função como g (x) = x / (1 / sin (pix)) = xsin (pix) Lembrando que csc (x) = 1 / sin (x). Agora, para localizar valores mínimos em um intervalo, reconheça que eles podem ocorrer nos terminais do intervalo ou em quaisquer valores críticos que ocorram dentro do intervalo. Para encontrar os valores críticos dentro do intervalo, defina a derivada da função igual a 0. E, para diferenciar a função, teremos que usar a regra do produto. Aplicação da regra do produto Consulte Mais informação »

Como você encontra lim_ (xtooo) log (4 + 5x) - log (x-1)?

Como você encontra lim_ (xtooo) log (4 + 5x) - log (x-1)?

Lim_ (xtooo) log (4 + 5x) - log (x-1) = log (5) lim_ (xtooo) log (4 + 5x) - log (x-1) = lim_ (xtooo) log ((4 + 5x) ) / (x-1)) Usando a regra da cadeia: lim_ (xtooo) log ((4 + 5x) / (x-1)) = lim_ (utoa) log (lim_ (xtooo) (4 + 5x) / (x- 1)) lim_ (xtooo) (ax + b) / (cx + d) = a / c lim_ (xtooo) (5x + 4) / (x-1) = 5 lim_ (uto5) log (u) = log5 Consulte Mais informação »

Como você diferencia y = cos (pi / 2x ^ 2-pix) usando a regra da cadeia?

Como você diferencia y = cos (pi / 2x ^ 2-pix) usando a regra da cadeia?

-sin (pi / 2x ^ 2-pix) * (pix-pi) Primeiro, pegue a derivada da função externa, cos (x): -sin (pi / 2x ^ 2-pix). Mas você também tem que multiplicar isso pela derivada do que está dentro, (pi / 2x ^ 2-pix). Faça este termo por termo. A derivada de pi / 2x ^ 2 é pi / 2 * 2x = pix. A derivada de -pix é apenas -pi. Então a resposta é -sin (pi / 2x ^ 2-pix) * (pix-pi) Consulte Mais informação »

Qual é a antiderivada de (2 + x ^ 2) / (1 + x ^ 2)?

Qual é a antiderivada de (2 + x ^ 2) / (1 + x ^ 2)?

A Resposta é x + arctan (x) Primeira nota que: (2 + x ^ 2) / (1 + x ^ 2) pode ser escrito como (1 + 1 + x ^ 2) / (1 + x ^ 2) = 1 / (1 + x ^ 2) + (1 + x ^ 2) / (1 + x ^ 2) = 1 + 1 / (1 + x ^ 2) => int (2 + x ^ 2) / (1 + x ^ 2) dx = int [1 + 1 / (1 + x ^ 2)] dx = int [1] dx + int [1 / (1 + x ^ 2)] dx = x + int [1 / ( 1 + x ^ 2)] dx = A derivada do arctan (x) é 1 / (1 + x ^ 2). Isto implica que a antiderivada de 1 / (1 + x ^ 2) é arctan (x) E é nessa base que podemos escrever: int [1 + 1 / (1 + x ^ 2)] dx = x + arctan ( x) Assim, int (2 + x ^ 2) / (1 + x ^ 2) dx == int [1 + 1 / (1 + x ^ 2)] dx = x + arc Consulte Mais informação »

Qual é a equação paramétrica de uma elipse?

Qual é a equação paramétrica de uma elipse?

Aqui está um exemplo ... Você pode ter (nsin (t), mcos (t)) quando n! = M, e n e m não são iguais a 1. Isto é essencialmente porque: => x = nsin (t) => x ^ 2 = n ^ 2sin ^ 2 (t) => x ^ 2 / n ^ 2 = sin ^ 2 (t) => y = mcos (t) => y ^ 2 / m ^ 2 = cos ^ 2 (t) => x ^ 2 / n ^ 2 + y ^ 2 / m ^ 2 = sen ^ 2 (t) + cos ^ 2 (t) Usando o fato de que sin ^ 2 (x) + cos ^ 2 ( x) = 1 ... => x ^ 2 / n ^ 2 + y ^ 2 / m ^ 2 = 1 Esta é essencialmente uma elipse! Note que se você quer uma elipse não circular, você tem que ter certeza que n! = M Consulte Mais informação »

Como você avalia a integral de int (cosx) / (sin ^ (2) x) dx?

Como você avalia a integral de int (cosx) / (sin ^ (2) x) dx?

Intcosx / sin ^ 2xdx = -cscx Deixe u = sinx, du = cosxdx e intcosx / sin ^ 2xdx = int (du) / u ^ 2 = -1 / u = -1 / sinx = -cscx Consulte Mais informação »

Como você encontra a velocidade instantânea em t = 2 para a função de posição s (t) = t ^ 3 + 8t ^ 2-t?

Como você encontra a velocidade instantânea em t = 2 para a função de posição s (t) = t ^ 3 + 8t ^ 2-t?

43 A velocidade instantânea é dada por (ds) / dt. Como s (t) = t ^ 3 + 8t ^ 2-t, (ds) / dt = 3t ^ 2 + 16t-1. Em t = 2, [(ds) / dt] _ (t = 2) = 3 * 2 ^ 2 + 16 * 2-1 = 43. Consulte Mais informação »

Como determinar a convergência ou divergência da sequência an = ln (n ^ 2) / n?

Como determinar a convergência ou divergência da sequência an = ln (n ^ 2) / n?

A sequência converge Para descobrir se a sequência a_n = ln (n ^ 2) / n = (2ln (n)) / n converge, observamos o que a_n é como n-> oo. n lim (n-> oo) a_n = lim_ (n-> oo) (2ln (n)) / n Usando a regra de l'Hôpital, = lim_ (n-> oo) (2 / n) / 1 = lim_ (n-> oo) 2 / n = 0 Como lim_ (n-> oo) a_n é um valor finito, a seqüência converge. Consulte Mais informação »

Como você diferencia f (x) = (x ^ 3-3x) (2x ^ 2 + 3x + 5) usando a regra do produto?

Como você diferencia f (x) = (x ^ 3-3x) (2x ^ 2 + 3x + 5) usando a regra do produto?

A resposta é (3x ^ 2-3) * (2x ^ 2 + 3x + 5) + (x ^ 3 - 3x) * (4x + 3), o que simplifica para 10x ^ 4 + 12x ^ 3-3x ^ 2- 18x-15. De acordo com a regra do produto, (f g) f = f ′ g + f g ′ Isto significa apenas que quando você diferencia um produto, você faz derivativo do primeiro, deixa o segundo sozinho, mais o derivativo do segundo, deixa o primeiro sozinho. Então o primeiro seria (x ^ 3 - 3x) e o segundo seria (2x ^ 2 + 3x + 5). Ok, agora a derivada da primeira é 3x ^ 2-3, vezes a segunda é (3x ^ 2-3) * (2x ^ 2 + 3x + 5). A derivada do segundo é (2 * 2x + 3 + 0), ou apenas (4x + 3). Mul Consulte Mais informação »

Pergunta # c76e4

Pergunta # c76e4

112pi "ou" 351,86 cm "/" min Uma moeda pode ser vista como um pequeno cilindro. E seu volume é obtido a partir da fórmula: V = pir ^ 2h Somos solicitados a descobrir como o volume está mudando. Isso significa que estamos olhando a taxa de variação de volume em relação ao tempo, ou seja, (dV) / (dt) Então tudo o que temos a fazer é diferenciar volume em relação ao tempo, como mostrado abaixo, => (dV) / (dt) = d (pir ^ 2h) / (dt) = pi (2r * (dr) / (dt) + (dh) / (dt)) Informamos que: (dr) / (dt) = 6 cm "/" min, (dh) / (dt) = 4 cm "/&q Consulte Mais informação »

Qual é a derivada de y = sec (2x) tan (2x)?

Qual é a derivada de y = sec (2x) tan (2x)?

2 seg (2x) (seg ^ 2 (2x) + tan ^ 2 (2x)) y '= (seg (2x)) (tan (2x))' + (tan (2x)) (seg (2x)) '( Regra do Produto) y '= (seg (2x)) (seg ^ 2 (2x)) (2) + (tan (2x)) (seg (2x) tan (2x)) (2) (Regra da cadeia e derivadas do trig ) y '= 2sec ^ 3 (2x) + 2seg (2x) tan ^ 2 (2x) y' = 2seg (2x) (seg ^ 2 (2x) + tan ^ 2 (2x)) Consulte Mais informação »

Qual é a regra do produto para derivativos? + Exemplo

Qual é a regra do produto para derivativos? + Exemplo

A regra do produto para derivados afirma que dada uma função f (x) = g (x) h (x), a derivada da função é f '(x) = g' (x) h (x) + g (x) h '(x) A regra do produto é usada principalmente quando a função para a qual se deseja a derivada é flagrantemente o produto de duas funções, ou quando a função seria mais facilmente diferenciada se vista como o produto de duas funções. Por exemplo, ao olhar para a função f (x) = tan ^ 2 (x), é mais fácil expressar a função como um produto, neste caso, sendo f (x) = tan (x) t Consulte Mais informação »

Como você encontra os derivados de y = (5x-2) ^ 3 (6x + 1) ^ 2 por diferenciação logarítmica?

Como você encontra os derivados de y = (5x-2) ^ 3 (6x + 1) ^ 2 por diferenciação logarítmica?

Y '= (5x-2) ^ 3 (6x + 1) ^ 2 ((15) / (5x-2) + (12) / (6x + 1)) 1 / ln (y) = 3ln (5x-2 ) + 2ln (6x + 1) 2 / (1) / (y) y '= (3) ((1) / (5x-2)) (5) + (2) ((1) / (6x + 1 )) (6) 3 / (1) / (y) y '= (15) / (5x-2) + (12) / (6x + 1) 4 / y' = y ((15) / (5x- 2) + (12) / (6x + 1)) 5 / y '= (5x-2) ^ 3 (6x + 1) ^ 2 ((15) / (5x-2) + (12) / (6x + 1)) Consulte Mais informação »

Qual é o propósito de um limite no cálculo?

Qual é o propósito de um limite no cálculo?

Um limite nos permite examinar a tendência de uma função em torno de um determinado ponto, mesmo quando a função não está definida no ponto. Vamos ver a função abaixo. f (x) = {x ^ 2-1} / {x-1} Como o seu denominador é zero quando x = 1, f (1) é indefinido; entretanto, seu limite em x = 1 existe e indica que o valor da função se aproxima de 2 lá. lim_ {x a 1} {x ^ 2-1} / {x-1} = lim_ {x a 1} {(x + 1) (x-1)} / {x-1} = lim_ {x a 1 } (x + 1) = 2 Esta ferramenta é muito útil no cálculo quando a inclinação de uma linha tangente é Consulte Mais informação »

Como você encontra a equação de uma linha tangente à função y = x ^ 2-5x + 2 em x = 3?

Como você encontra a equação de uma linha tangente à função y = x ^ 2-5x + 2 em x = 3?

Y = x-7 Seja y = f (x) = x ^ 2-5x + 2 Em x = 3, y = 3 ^ 2-5 * 3 + 2 = 9-15 + 2 = -6 + 2 = -4 Então, a coordenada está em (3, -4). Primeiro precisamos encontrar o declive da linha tangente no ponto, diferenciando f (x), e conectando x = 3 lá. : .f '(x) = 2x-5 Em x = 3, f' (x) = f '(3) = 2 * 3-5 = 6-5 = 1 Assim, a inclinação da linha tangente será 1. Agora, usamos a fórmula de declive do ponto para descobrir a equação da linha, ou seja: y-y_0 = m (x-x_0) onde m é a inclinação da linha, (x_0, y_0) são o original coordenadas. E assim, y - (- 4) = 1 (x Consulte Mais informação »

Qual é a taxa de variação da largura (em ft / s) quando a altura é de 10 pés, se a altura estiver diminuindo nesse momento a uma taxa de 1 pé / seg.Um retângulo tem uma altura variável e uma largura variável , mas a altura e a largura mudam para que a área do retângulo seja sempre de 60 pés quadrados?

Qual é a taxa de variação da largura (em ft / s) quando a altura é de 10 pés, se a altura estiver diminuindo nesse momento a uma taxa de 1 pé / seg.Um retângulo tem uma altura variável e uma largura variável , mas a altura e a largura mudam para que a área do retângulo seja sempre de 60 pés quadrados?

A taxa de variação da largura com o tempo (dW) / (dt) = 0,6 "ft / s" (dW) / (dt) = (dW) / (dh) xx (dh) / dt (dh) / (dt ) = - 1 "ft / s" Assim (dW) / (dt) = (dW) / (dh) xx-1 = - (dW) / (dh) Wxxh = 60 W = 60 / h (dW) / ( dh) = - (60) / (h ^ 2) Então (dW) / (dt) = - (- (60) / (h ^ 2)) = (60) / (h ^ 2) Então quando h = 10 : rArr (dW) / (dt) = (60) / (10 ^ 2) = 0,6 "ft / s" Consulte Mais informação »

Qual é a relação entre a taxa média de mudança de uma função e derivados?

Qual é a relação entre a taxa média de mudança de uma função e derivados?

A taxa média de mudança dá a inclinação de uma linha secante, mas a taxa instantânea de mudança (a derivada) dá a inclinação de uma linha tangente. Taxa de variação média: (f (x + h) -f (x)) / h = (f (b) -f (a)) / (ba), em que o intervalo é [a, b] Taxa de alteração instantânea : lim_ (h -> 0) (f (x + h) -f (x)) / h Observe também que a taxa média de mudança se aproxima da taxa instantânea de mudança em intervalos muito curtos. Consulte Mais informação »

Qual é o máximo relativo de y = csc (x)?

Qual é o máximo relativo de y = csc (x)?

Y = cscx = 1 / sinx = (sinx) ^ - 1 Para encontrar um max / min, encontramos a primeira derivada e encontramos os valores para os quais a derivada é zero. y = (senx) ^ - 1: .y '= (- 1) (senx) ^ - 2 (cosx) (regra da cadeia): .y' = - cosx / sin ^ 2x No max / min, y '= 0 => - cosx / sin ^ 2x = 0: .cosx = 0: .x = -pi / 2, pi / 2, ... Quando x = pi / 2 => y = 1 / sin (pi / 2) = 1 Quando x = -pi / 2 => y = 1 / sin (-pi / 2) = - 1 Então há pontos de virada em (-pi / 2, -1) e (pi / 2,1) Se olharmos no gráfico de y = cscx, observamos que (-pi / 2, -1) é um máximo relativo e (pi / 2, Consulte Mais informação »

Como você encontra a integral indefinida de x ^ 2 - 2 dx / x ^ 3 - 4x?

Como você encontra a integral indefinida de x ^ 2 - 2 dx / x ^ 3 - 4x?

I = 1 / 4ln (x ^ 4-4x ^ 2) + C Queremos resolver I = int (x ^ 2-2) / (x ^ 3-4x) dx Multiplique o DEN e o NUM por x I = int ( x ^ 3-2x) / (x ^ 4-4x ^ 2) dx Agora podemos fazer uma cor substituta legal (vermelho) (u = x ^ 4-4x ^ 2 => du = 4x ^ 3-8xdx = 4 ( x ^ 3-2x) dx I = 1 / 4int1 / udu cor (branco) (I) = 1 / 4ln (u) + C cor (branco) (I) = 1 / 4ln (x ^ 4-4x ^ 2) + C Consulte Mais informação »

Qual é a operação de gradiente reverso?

Qual é a operação de gradiente reverso?

Conforme explicado abaixo. Se houver um campo vetorial conservador F (x, y, z) = Mdx + Ndy + Pdz. sua função potencial pode ser encontrada. Se a função potencial é, digamos, f (x, y, z), então f_x (x, y, z) = M, f_y (x, y, z) = N e f_z (x, y, z) = P . Então, f (x, y, z) = int Mdx + C1 f (x, y, z) = int Nd + C2 e f (x, y, z) = int Pdz + C3, Onde C1 seria alguma função de y e z, C2 seria alguma função de x e z, C3 seria alguma função de x e y A partir dessas três versões de f (x, y, z), a função potencial f (x, y, z) pode ser detectada . Lev Consulte Mais informação »

Qual é a derivada de arcsin (1 / x)?

Qual é a derivada de arcsin (1 / x)?

-1 / (xsqrt (x ^ 2-1)) Para diferenciar isso, aplicaremos uma regra de cadeia: Comece por Deixar teta = arcsin (1 / x) => sin (teta) = 1 / x Agora diferencie cada termo em ambos os lados da equação com respeito a x => cos (theta) * (d (teta)) / (dx) = - 1 / x ^ 2 Usando a identidade: cos ^ 2theta + sin ^ 2theta = 1 => costheta = sqrt (1-sin ^ 2theta) => sqrt (1-sin ^ 2theta) * (d (teta)) / (dx) = - 1 / x ^ 2 => (d (teta)) / (dx) = - 1 / x ^ 2 * 1 / sqrt (1-sin ^ 2theta) Lembre-se: sin (theta) = 1 / x "" e "" theta = arcsin (1 / x) Assim, podemos escrever, (d (arcsin (1 / x))) / Consulte Mais informação »

Qual é a segunda derivada de 1 / x ^ 2?

Qual é a segunda derivada de 1 / x ^ 2?

F '' (x) = 6 / x ^ 4> reescreva f (x) = 1 / x ^ 2 = x ^ -2 rArr f '(x) = -2x ^ -3 rArr f' '(x) = 6x ^ -4 = 6 / x ^ 4 Consulte Mais informação »

Qual é a segunda derivada de (f * g) (x) se f e g são funções tais que f '(x) = g (x) e g' (x) = f (x)?

Qual é a segunda derivada de (f * g) (x) se f e g são funções tais que f '(x) = g (x) e g' (x) = f (x)?

(4f * g) (x) Seja P (x) = (f * g) (x) = f (x) g (x) Então, usando a regra do produto: P '(x) = f' (x) g ( x) + f (x) g '(x). Usando a condição dada na pergunta, obtemos: P '(x) = (g (x)) ^ 2+ (f (x)) ^ 2 Agora, usando as regras de potência e de cadeia: P' '(x) = 2g (x) g '(x) + 2f (x) f' (x). Aplicando novamente a condição especial desta questão, escrevemos: P '' (x) = 2g (x) f (x) + 2f (x) g (x) = 4f (x) g (x) = 4 (f * g) (x) Consulte Mais informação »

Qual é a segunda derivada de g (x) = sec (3x + 1)?

Qual é a segunda derivada de g (x) = sec (3x + 1)?

H '' (x) = 9 seg (3x + 1) [seg ^ 2 (3x + 1) + tan ^ 2 (3x + 1)] Dado: h (x) = seg (3x + 1) Use o seguinte derivativo regras: (sec u) '= u' sec u tan u; "" (tan u) '= u' sec ^ 2 u Regra do produto: (fg) '= f g' + g f 'Encontre a primeira derivada: Deixe u = 3x + 1; "" u '= 3 h' (u) = 3 seg u u u uh '(x) = 3 seg (3x + 1) tan (3x + 1) Encontre a segunda derivada usando a regra do produto: Seja f = 3 seg (3x + 1); "" f '= 9 seg (3x + 1) tan (3x + 1) Deixe g = tan (3x + 1); "" g '= 3 seg ^ 2 (3x + 1) h' '(x) = (3 seg (3x + 1) Consulte Mais informação »

Qual é a segunda derivada da função f (x) = sec x?

Qual é a segunda derivada da função f (x) = sec x?

F '' (x) = sec x ( sec ^ 2 x + tan ^ 2 x) função dada: f (x) = sec x Diferenciação w.r.t. x da seguinte forma frac {d} {dx} f (x) = frac {d} {dx} ( seg x) f '(x) = sec x tan x Novamente, diferenciando f' (x) w.r.t. x, obtemos frac {d} {dx} f '(x) = frac {d} {dx} ( sec x tan x) f' '(x) = sec x frac {d} { dx} tan x + tan x frac {d} {dx} secx = seg xsec ^ 2 x + tan x seg x tan x = seg ^ 3 x + seg x tan ^ 2 x = seg x ( sec ^ 2 x + tan ^ 2 x) Consulte Mais informação »

Qual é a segunda derivada da função f (x) = (x) / (x - 1)?

Qual é a segunda derivada da função f (x) = (x) / (x - 1)?

D ^ 2 / (dx ^ 2) x / (x-1) = 2 / (x-1) ^ 3 Para este problema, usaremos a regra do quociente: d / dx f (x) / g (x) = (g (x) f '(x) -f (x) g' (x)) / [g (x)] ^ 2 Nós também podemos tornar isto um pouco mais fácil dividindo para obter x / (x-1) = 1 + 1 / (x-1) Primeira derivada: d / dx (1 + 1 / (x-1)) = (d / dx1) + (d / dx ((x-1) (d / dx1) -1 (d / dx (x-1))) / (x-1) ^ 2) = 0 + ((x-1) (0) - (1) (1)) / (x-1) ^ 2 = - 1 / (x-1) ^ 2 Segunda derivada: A segunda derivada é a derivada da primeira derivada. d ^ 2 / (dx ^ 2) (1 + 1 / (x-1)) = d / dx (-1 / (x-1) ^ 2) = - ((x-1) ^ 2 (d / dx1 ) -1 (d / dx (x-1) Consulte Mais informação »

Qual é a segunda derivada de x / (x-1) e a primeira derivada de 2 / x?

Qual é a segunda derivada de x / (x-1) e a primeira derivada de 2 / x?

Questão 1 Se f (x) = (g (x)) / (h (x)) então pela Regra do Quociente f '(x) = (g' (x) * h (x) - g (x) * h '(x)) / ((g (x)) ^ 2) Então se f (x) = x / (x-1) então a primeira derivada f' (x) = ((1) (x-1) - (x) (1)) / x ^ 2 = - 1 / x ^ 2 = - x ^ (- 2) e a segunda derivada é f '' (x) = 2x ^ -3 Pergunta 2 Se f (x) = 2 / x isso pode ser reescrito como f (x) = 2x ^ -1 e usando procedimentos padrão para obter a derivada f '(x) = -2x ^ -2 ou, se você preferir f' (x) = - 2 / x ^ 2 Consulte Mais informação »

Qual é a segunda derivada de y = x * sqrt (16-x ^ 2)?

Qual é a segunda derivada de y = x * sqrt (16-x ^ 2)?

Y ^ ('') = (2 * x (x ^ 2 - 24)) / ((16-x ^ 2) * sqrt (16-x ^ 2)) Comece calculando a primeira derivada da sua função y = x * sqrt (16-x ^ 2) usando a regra do produto. Isto fará com que você d / dx (y) = [d / dx (x)] * sqrt (16 - x ^ 2) + x * d / dx (sqrt (16 - x ^ 2)) Você pode diferenciar d / dx (sqrt (16 -x ^ 2)) usando a regra da cadeia para sqrt (u), com u = 16 -x ^ 2. d / dx (sqrt (u)) = d / (du) sqrt (u) * d / dx (u) d / dx (sqrt (u)) = 1/2 * 1 / sqrt (u) * d / dx (16-x ^ 2) d / dx (sqrt (16-x ^ 2)) = 1 / cor (vermelho) (cancelar (cor (preto) (2))) * 1 / sqrt (16-x ^ 2) * (-color (ve Consulte Mais informação »

Como você integra int 1 / (x ^ 2 (2x-1)) usando frações parciais?

Como você integra int 1 / (x ^ 2 (2x-1)) usando frações parciais?

2ln | 2x-1 | -2ln | x | + 1 / x + C Precisamos encontrar A, B, C tal que 1 / (x ^ 2 (2x-1)) = A / x + B / x ^ 2 + C / (2x-1) para todo x. Multiplique ambos os lados por x ^ 2 (2x-1) para obter 1 = Ax (2x-1) + B (2x-1) + Cx ^ 2 1 = 2Ax ^ 2-Ax + 2Bx-B + Cx ^ 2 1 = (2A + C) x ^ 2 + (2B-A) xB Os coeficientes de equação nos dão {(2A + C = 0), (2B-A = 0), (- B = 1):} E assim temos A = -2, B = -1, C = 4. Substituindo isso na equação inicial, obtemos 1 / (x ^ 2 (2x-1)) = 4 / (2x-1) -2 / x-1 / x ^ 2 Agora, integre-o termo por termo int 4 / (2x-1) dx-int 2 / x dx-int 1 / x ^ 2 dx para obter 2ln | 2x-1 | -2ln Consulte Mais informação »

Calcular o valor aproximado de int_0 ^ 6x ^ 3 dx tomando 6 subintervalos de igual duração e aplicando a regra de Simpson?

Calcular o valor aproximado de int_0 ^ 6x ^ 3 dx tomando 6 subintervalos de igual duração e aplicando a regra de Simpson?

Int_0 ^ 6x ^ 3dx ~~ 324 A regra de Simpson diz que int_b ^ af (x) dx pode ser aproximado por h / 3 [y_0 + y_n + 4y_ (n = "ímpar") + 2y_ (n = "par") h = (ba) / n = (6-0) / 6 = 6/6 = 1 int_0 ^ 6x ^ 3dx ~~ 1/3 [0 + 216 + 4 (1 + 27 + 125) +2 (8 + 64)] = [216 + 4 (153) +2 (72)] / 3 = [216 + 612 + 144] = 972/3 = 324 Consulte Mais informação »

Como encontro a convergência ou divergência desta série? soma de 1 a infinito de 1 / n ^ lnn

Como encontro a convergência ou divergência desta série? soma de 1 a infinito de 1 / n ^ lnn

Converge Considere a série sum_ (n = 1) ^ oo1 / n ^ p, onde p> 1. Pelo teste p, esta série converge. Agora, 1 / n ^ ln n <1 / n ^ p para todos os grandes o suficiente n contanto que p seja um valor finito. Assim, pelo teste de comparação direta, sum_ (n = 1) ^ oo1 / n ^ ln n converge. De fato, o valor é aproximadamente igual a 2.2381813. Consulte Mais informação »

Qual é a derivada de y = (sinx) ^ x?

Qual é a derivada de y = (sinx) ^ x?

Dy / dx = (ln (sinx) + xcotx) (sinx) ^ x Use diferenciação logarítmica. y = (sinx) ^ x lny = ln ((sinx) ^ x) = xln (sinx) (Use propriedades de ln) Diferencie implicitamente: (Use a regra do produto e a cadeia ruel) 1 / y dy / dx = 1ln ( sinx) + x [1 / sinx cosx] Então, temos: 1 / y dy / dx = ln (senx) + x cotx Resolva para dy / dx multiplicando por y = (senx) ^ x, dy / dx = ( ln (sinx) + xcotx) (sinx) ^ x Consulte Mais informação »

Como você encontra a derivada de f (x) = [(2x-5) ^ 5] / [(x ^ 2 +2) ^ 2] usando a regra da cadeia?

Como você encontra a derivada de f (x) = [(2x-5) ^ 5] / [(x ^ 2 +2) ^ 2] usando a regra da cadeia?

= (10 (2x-5) ^ 4 * (x ^ 2 + 2) ^ 2 - (2x-5) ^ 5 * 4x (x ^ 2 + 2)) / (x ^ 2 + 2) ^ 4 f ' (x) = (f '(x) * g (x) - f (x) * g' (x)) / (g (x)) ^ 2 f '(x) = (((5 (2x-5 ) ^ 4 * 2) (x ^ 2 + 2) ^ 2) - (2x-5) ^ 5 * (2 (x ^ 2 + 2) * 2x)) / ((x ^ 2 + 2) ^ 2) ^ 2 = (10 (2x-5) ^ 4 * (x ^ 2 + 2) ^ 2 - (2x-5) ^ 5 * 4x (x ^ 2 + 2)) / (x ^ 2 + 2) ^ 4 Você pode reduzir mais, mas está entediado resolver essa equação, basta usar o método algébrico. Consulte Mais informação »

Como você diferencia o sqrt (cos (x ^ 2 + 2)) + sqrt (cos ^ 2x + 2)?

Como você diferencia o sqrt (cos (x ^ 2 + 2)) + sqrt (cos ^ 2x + 2)?

(dy) / (dx) = (xsen (x ^ 2 + 2) + sen (x + 2)) / (sqrtcos (x ^ 2 + 2) + sqrt (cos ^ 2 (x + 2))) (dy ) / (dx) = 1 / (2sqrtcos (x ^ 2 + 2) + sqrt (cos ^ 2 (x + 2))) * sen (x ^ 2 + 2) * 2x + 2sen (x + 2) (dy ) / (dx) = (2xsen (x ^ 2 + 2) + 2sen (x + 2)) / (2sqrtcos (x ^ 2 + 2) + sqrt (cos ^ 2 (x + 2))) (dy) / (dx) = (cancel2 (xsen (x ^ 2 + 2) + sen (x + 2))) / (cancel2sqrtcos (x ^ 2 + 2) + sqrt (cos ^ 2 (x + 2))) (dy) / (dx) = (xsen (x ^ 2 + 2) + sen (x + 2)) / (sqrtcos (x ^ 2 + 2) + sqrt (cos ^ 2 (x + 2))) Consulte Mais informação »

Como você encontra os três primeiros termos de uma série Maclaurin para f (t) = (e ^ t - 1) / t usando a série Maclaurin de e ^ x?

Como você encontra os três primeiros termos de uma série Maclaurin para f (t) = (e ^ t - 1) / t usando a série Maclaurin de e ^ x?

Sabemos que a série Maclaurin de e ^ x é sum_ (n = 0) ^ oox ^ n / (n!) Também podemos derivar esta série usando a expansão Maclaurin de f (x) = sum_ (n = 0) ^ oof ^ ((n)) (0) x ^ n / (n!) e o fato de que todas as derivadas de e ^ x ainda são e ^ xe e ^ 0 = 1. Agora, apenas substitua a série acima em (e ^ x-1) / x = (soma_ (n = 0) ^ oo (x ^ n / (n!)) - 1) / x = (1 + soma_ (n = 1) ^ oo (x ^ n / (n!)) - 1) / x = (soma_ (n = 1) ^ oo (x ^ n / (n!))) / X = sum_ (n = 1) ^ oox ^ (n-1) / (n!) Se você quiser que o índice comece em i = 0, simplesmente substitua n = i + 1: = sum_ (i = 0) ^ Consulte Mais informação »

Qual é a inclinação da curva polar f (teta) = teta - sec ^ 3theta + thetasin ^ 3theta em teta = (5pi) / 8?

Qual é a inclinação da curva polar f (teta) = teta - sec ^ 3theta + thetasin ^ 3theta em teta = (5pi) / 8?

Dy / dx = -0,54 Para uma função polar f (teta), dy / dx = (f '(teta) sineta + f (teta) costheta) / (f' (teta) costheta-f (teta) sineta) f ( theta) = teta-sec ^ 3theta + thetasin ^ 3theta f '(theta) = 1-3 (sec ^ 2theta) (d / dx [sectheta]) - sin ^ 3theta + 3thetasin ^ 2theta (d / dx [sintheta]) f '(theta) = 1-3sec ^ 3thetatantheta-sin ^ 3theta + 3thetasin ^ 2thetacostheta f' ((5pi) / 3) = 1-3sec ^ 3 ((5pi) / 3) tan ((5pi) / 3) - sen ^ 3 ((5pi) / 3) +3 ((5pi) / 3) sen ^ 2 ((5pi) / 3) cos ((5pi) / 3) ~ ~ -9,98 f ((5pi) / 3) = ((5pi) / 3) -sec ^ 3 ((5pi) / 3) + ((5pi) / 3) sin ^ 3 ((5pi) / 3) ~~ - Consulte Mais informação »

Como posso encontrar a derivada de y = (x ^ 2 + 1) ^ 5?

Como posso encontrar a derivada de y = (x ^ 2 + 1) ^ 5?

Dy / dx = 10x (x ^ 2 + 1) ^ 4 Se escrevermos isto como: y = u ^ 5 então podemos usar a regra da cadeia: dy / dx = (dy) / (du) * (du) / ( dx) (d) / (du) = 5u ^ 4 (du) / (dx) = 2x dy / dx = (d) / (du) * (du) / (dx) = 10xu ^ 4 Colocando de volta em x ^ 2 + 1 nos dá: dy / dx = 10x (x ^ 2 + 1) ^ 4 Consulte Mais informação »

Qual é a inclinação da linha tangente ao gráfico da função f (x) = ln (sen ^ 2 (x + 3)) no ponto em que x = pi / 3?

Qual é a inclinação da linha tangente ao gráfico da função f (x) = ln (sen ^ 2 (x + 3)) no ponto em que x = pi / 3?

Ver abaixo. Se: y = lnx <=> e ^ y = x Usando esta definição com determinada função: e ^ y = (sin (x + 3)) ^ 2 Diferenciando implicitamente: e ^ ydy / dx = 2 (sin (x + 3) )) * cos (x + 3) Dividindo por e ^ y dy / dx = (2 (sen (x + 3)) * cos (x + 3)) / e ^ y dy / dx = (2 (sen (x +3)) * cos (x + 3)) / (sen ^ 2 (x + 3)) Cancelando fatores comuns: dy / dx = (2 (cancelar (sen (x + 3))) * cos (x + 3 )) / (sen ^ cancelar (2) (x + 3)) dy / dx = (2cos (x + 3)) / (sen (x + 3)) Agora temos a derivada e, portanto, seremos capazes de calcular a gradiente em x = pi / 3 Conectando este valor: (2cos ((pi / 3) +3)) Consulte Mais informação »

Precisa de ajuda com esta equação limite? lim_ (x 0 ^ +) x ^ 4 ln (x)

Precisa de ajuda com esta equação limite? lim_ (x 0 ^ +) x ^ 4 ln (x)

Lim_ (xto0 ^ +) x ^ 4ln (x) = 0 f (x) = x ^ 4ln (x) [(x, f (x)), (1,0), (0,1, -2,30 * 10 ^ - 4), (0.01, -4.61 * 10 ^ -8), (0.001, -6.91 * 10 ^ -12)] Como x tende a 0 do lado direito, f (x) permanece no lado negativo quando x < 1, mas os próprios valores se aproximam de 0 quando x-> 0 lim_ (xto0 ^ +) x ^ 4ln (x) = 0 gráfico {x ^ 4ln (x) [-0,05 1, -0,1, 0,01]} Consulte Mais informação »

Qual é a inclinação da linha tangente para a equação y = x ^ 2 (3x + 1 / x ^ 3) em x = 1/3?

Qual é a inclinação da linha tangente para a equação y = x ^ 2 (3x + 1 / x ^ 3) em x = 1/3?

A inclinação da tangente para y em x = 1/3 é -8 y = x ^ 2 (3x + 1 / x ^ 3) = x ^ 2 (3x + x ^ (- 3)) dy / dx = x ^ 2 ( 3-3x ^ (- 4)) + 2x (3x + x ^ (- 3)) Regra do produto = 3x ^ 2-3x ^ (- 2) + 6x ^ 2 + 2x ^ (- 2) = 9x ^ 2- x ^ (- 2) A inclinação (m) da tangente para y em x = 1/3 é dy / dx em x = 1/3 Assim: m = 9 * (1/3) ^ 2 - (1/3 ) ^ (- 2) m = 1-9 = 8 Consulte Mais informação »

Qual é a inclinação da linha tangente no mínimo de uma curva suave?

Qual é a inclinação da linha tangente no mínimo de uma curva suave?

A inclinação é 0. Mínimos (o plural de 'mínimo') de curvas suaves ocorrem em pontos de virada, que por definição também são pontos estacionários. Estes são chamados de estacionários porque nesses pontos, a função de gradiente é igual a 0 (então a função não está "em movimento", ou seja, é estacionária).Se a função gradiente é igual a 0, então a inclinação da linha tangente nesse ponto também é igual a 0. Um exemplo fácil para a imagem é y = x ^ 2. Consulte Mais informação »

Como eu resolvo esse limite?

Como eu resolvo esse limite?

E ^ a * (a / 2) * (1 - a) "Você pode usar as séries de Taylor e descartar termos de ordem mais alta no" "limite de" x-> 0 "." x ^ y = exp (y * ln (x)) => (1 + x) ^ y = exp (y * ln (1 + x)) "e" ln (1 + x) = x - x ^ 2 / 2 + x ^ 3/3 - ... "e" exp (x) = 1 + x + x ^ 2/2 + x ^ 3/6 + x ^ 4/24 + ... "Então" exp (y * ln (1 + x)) = exp (y * (x - x ^ 2/2 + ...)) => (1 + x) ^ (a / x) = exp ((a / x) * ln (1 + x)) = exp ((a / x) * (x - x ^ 2/2 + x ^ 3/3 - ...)) = exp (a - a * x / 2 + a * x ^ 2/3 - ...) => (1 + ax) ^ (1 / x) = exp ((1 / x) * ln (1 + ax Consulte Mais informação »

Como você usa a regra trapezoidal com n = 4 para aproximar a área entre a curva 1 / (1 + x ^ 2) de 0 a 6?

Como você usa a regra trapezoidal com n = 4 para aproximar a área entre a curva 1 / (1 + x ^ 2) de 0 a 6?

Use a fórmula: Área = h / 2 (y_1 + y_n + 2 (y_2 + y_3 + ... + y_ (n-1))) para obter o resultado: Área = 4314/3145 ~ = 1,37 h é o comprimento do passo encontre o comprimento do passo usando a seguinte fórmula: h = (ba) / (n-1) a é o valor mínimo de x e b é o valor máximo de x. No nosso caso a = 0 e b = 6 n é o número de tiras. Daí n = 4 => h = (6-0) / (4-1) = 2 Assim, os valores de x são 0,2,4,6 "NB:" A partir de x = 0 adicionamos o comprimento do passo h = 2 para obter o próximo valor de x até x = 6 A fim de encontrar y_1 até y_n ( Consulte Mais informação »

Por favor ajude!!! esta é uma escolha múltipla. determine o valor mínimo da função f (x) = e ^ (- x) -2e ^ x no intervalo -1 x 2.

Por favor ajude!!! esta é uma escolha múltipla. determine o valor mínimo da função f (x) = e ^ (- x) -2e ^ x no intervalo -1 x 2.

A resposta é o mínimo no intervalo é f (2) = e ^ 2} -2e ^ 2, o que não é realmente uma escolha, mas (c) é uma boa aproximação. f (x) = e ^ x} - 2e ^ x f '(x) = - e ^ x} - 2 e ^ x Essa derivada é claramente negativa em todos os lugares, então a função está diminuindo ao longo do intervalo. Portanto, seu valor mínimo é f (2) = e ^ 2} -2e ^ 2. Se eu fosse um defensor (o que sou), eu responderia a Nenhum dos Acima porque não há como a quantidade transcendental poder se igualar a um desses valores racionais. Mas nós sucumbimos à c Consulte Mais informação »

Encontre a equação da tangente à curva y = 2- x perpendicular à reta y + 4x-4 = 0?

Encontre a equação da tangente à curva y = 2- x perpendicular à reta y + 4x-4 = 0?

A inclinação da perpendicular é 1/4, mas a derivada da curva é -1 / {2sqrt {x}}, que será sempre negativa, portanto a tangente à curva nunca é perpendicular a y + 4x = 4. f (x) = 2 - x ^ {1/2} f '(x) = - 1/2 x ^ {- 1/2} = -1 / {2sqrt {x}} A linha dada é y = -4x + 4 então tem inclinação -4, então suas perpendiculares têm a inclinação recíproca negativa, 1/4. Nós definimos a derivada igual a isso e resolvemos: 1/4 = -1 / {2 sqrt {x}} sqrt {x} = -2 Não há nenhum x real que satisfaça isso, então nenhum lugar na curva on Consulte Mais informação »

A série é absolutamente convergente, condicionalmente convergente ou divergente? rarr 4-1 + 1 / 4-1 / 16 + 1/64 ...

A série é absolutamente convergente, condicionalmente convergente ou divergente? rarr 4-1 + 1 / 4-1 / 16 + 1/64 ...

Converge absolutamente. Use o teste para convergência absoluta. Se tomarmos o valor absoluto dos termos, obtemos a série 4 + 1 + 1/4 + 1/16 + ... Esta é uma série geométrica de razão comum 1/4. Assim converge. Já que ambos | a_n | Converge a_n converge absolutamente. Espero que isso ajude! Consulte Mais informação »

Como encontrar h em termos de x?

Como encontrar h em termos de x?

H = 1000 / (2pix) - x para 31a, você precisa da fórmula para a área de superfície total de um cilindro. a área total da superfície de um cilindro é a mesma do total de ambas as superfícies circulares (superior e inferior) e da área de superfície curva. a área de superfície curva pode ser considerada como um retângulo (se for para ser lançado). o comprimento desse retângulo seria a altura do cilindro e sua largura seria a circunferência de um círculo na parte superior ou inferior. a circunferência de um círculo é 2pir. altura Consulte Mais informação »

Pergunta # f9641

Pergunta # f9641

Int cos (x) / (sen ^ 2 (x) + sen (x)) "d" x = ln | sen (x) / (sin (x) +1) | + C int cos (x) / (sen ^ 2 (x) + sen (x)) "d" x Substitua u = sin (x) e "d" u = cos (x) "d" x. Isto dá = int ("d" u) / (u ^ 2 + u) = int ("d" u) / (u (u + 1)) Separado para frações parciais desde 1 / (u (u + 1 )) = 1 / u-1 / (u + 1): = int (1 / u-1 / (u + 1)) "d" u = ln | u | -ln | u + 1 | + C = ln | u / (u + 1) | + C Substituto de volta u = sin (x): = ln | sin (x) / (sin (x) +1) | + C Consulte Mais informação »

Como integrar o sqrt (x ^ 2 + 4x) dx?

Como integrar o sqrt (x ^ 2 + 4x) dx?

Int sqrt (x ^ 2 + 4x) dx = sinh (2cosh ^ -1 ((x + 2) / 2)) - 2cosh ^ -1 ((x + 2) / 2) + C Como é mais fácil lidar com apenas um x sob uma raiz quadrada, completamos o quadrado: x ^ 2 + 4x = (x + 2) ^ 2 + kx ^ 2 + 4x = x ^ 2 + 4x + 4 + kk = -4 x ^ 2 + 4x = (x + 2) ^ 2-4 int sqrt (x ^ 2 + 4x) dx = int sqrt ((x + 2) ^ 2-4) dx Agora precisamos fazer uma substituição trigonométrica. Eu vou usar funções trigonométricas hiperbólicas (porque integrais secantes geralmente não são muito boas). Queremos usar a seguinte identidade: cosh ^ 2 (teta) -1 = sinh ^ 2 (teta) Para fazer i Consulte Mais informação »

Em que intervalos a seguinte equação é côncava para cima, côncava para baixo e onde o ponto de inflexão é (x, y) f (x) = x ^ 8 (ln (x))?

Em que intervalos a seguinte equação é côncava para cima, côncava para baixo e onde o ponto de inflexão é (x, y) f (x) = x ^ 8 (ln (x))?

Se 0 <x <e ^ (- 15/56), então f é côncava para baixo; se x> e ^ (- 15/56) então f é côncavo para cima; x = e ^ (- 15/56) é um ponto de inflexão (queda) Para analisar os pontos de concavidade e inflexão de uma função f duas vezes diferenciável, podemos estudar a positividade da segunda derivada. De fato, se x_0 é um ponto no domínio de f, então: se f '' (x_0)> 0, então f é côncavo em uma vizinhança de x_0; se f '' (x_0) <0, então f é côncava para baixo em uma vizinhança de x_0; se Consulte Mais informação »

Em que intervalo é f (x) = 6x ^ 3 + 54x-9 côncavo para cima e para baixo?

Em que intervalo é f (x) = 6x ^ 3 + 54x-9 côncavo para cima e para baixo?

Uma função é côncava quando a segunda derivada é positiva, é côncava para baixo quando é negativa e pode haver um ponto de inflexão quando é zero. y '= 18x ^ 2 + 54 y' '= 36x + 54 e: y' '> 0rArrx> -54 / 36rArrx> -3/2. Em (-3 / 2, + oo) o côncavo é para cima, em (-oo, -3 / 2) o côncavo é para baixo, em x = -3 / 2 há um ponto de inflexão. Consulte Mais informação »

Como escolher dois números para os quais a soma de suas raízes quadradas é mínima, sabendo que o produto dos dois números é um?

Como escolher dois números para os quais a soma de suas raízes quadradas é mínima, sabendo que o produto dos dois números é um?

X = y = sqrt (a) x * y = a => x * y - a = 0 f (x, y) = sqrt (x) + sqrt (y) "é mínimo" "Podemos trabalhar com o multiplicador de Lagrange L: "f (x, y, L) = sqrt (x) + sqrt (y) + L (x * ya)" Derivando rendimentos: "{df} / dx = 1 / (2 * sqrt (x)) + L * y = 0 {df} / dy = 1 / (2 * sqrt (y)) + L * x = 0 {df} / {dL} = x * ya = 0 => y = a / x = & gt; df} / dy = 1 / (2 * sqrt (a / x)) + L * x = 0 = sqrt (x) / (2 * sqrt (a)) + L * x = 0 => {df} / dx = 1 / (2 * sqrt (x)) + L * a / x = 0 => sqrt (x) / 2 + L * a = 0 "(depois de multiplicar por x"! = "0)" => Consulte Mais informação »

Como resolver sem a regra do Hospital? lim_ (x-> 0) (xcos ^ 2 (x)) / (x + tan (3x))

Como resolver sem a regra do Hospital? lim_ (x-> 0) (xcos ^ 2 (x)) / (x + tan (3x))

1/4 "Você poderia usar a expansão da série de Taylor." cos (x) = 1 - x ^ 2/2! + x ^ 4/4! - ... tan (x) = x + x ^ 3/3 + 2 x ^ 5/15 + ... => cos ^ 2 (x) = 1 - x ^ 2 + x ^ 4 (1/4 + 2/24) ... = 1 - x ^ 2 + x ^ 4/3 ... => tan (3x) = 3x + 9 x ^ 3 + ... => (x * cos ^ 2 (x) ) / (x + tan (3x)) = (x - x ^ 3 + x ^ 5/3 ...) / (4x + 9 x ^ 3 + ...) x-> 0 => "maiores poderes desaparecem "= (x - ...) / (4x + ...) = 1/4 Consulte Mais informação »

Integração de 1 / (1 + x ^ 3) dx?

Integração de 1 / (1 + x ^ 3) dx?

1 / 3ln | x + 1 | -1 / 6ln | x ^ 2-x + 1 | + sqrt3 / 3tano ^ -1 ((2x-1) / sqrt3) + C Comece por fatorizar o denominador: 1 + x ^ 3 = (x + 1) (x ^ 2-x + 1) Agora podemos fazer frações parciais: 1 / (1 + x ^ 3) = 1 / ((x + 1) (x ^ 2-x + 1)) = A / (x + 1) + (Bx + C) / (x ^ 2-x + 1) Podemos encontrar A usando o método de encobrimento: A = 1 / ((text (////)) ( (-1) ^ 2 + 1 + 1)) = 1/3 A seguir podemos multiplicar ambos os lados pelo denominador LHS: 1 = 1/3 (x ^ 2-x + 1) + (Bx + C) (x + 1) 1 = 1 / 3x ^ 2-1 / 3x + 1/3 + Bx ^ 2 + Bx + Cx + C1 = (1/3 + B) x ^ 2 + (B + C-1/3) x + (C + 1/3) Isto dá as seguintes e Consulte Mais informação »

Determine a equação da linha tangente à curva definida por (2x ^ 4) (4y ^ 4) + 6x ^ 3 + 7y ^ 2 = 2703 no ponto (2, 3)?

Determine a equação da linha tangente à curva definida por (2x ^ 4) (4y ^ 4) + 6x ^ 3 + 7y ^ 2 = 2703 no ponto (2, 3)?

O ponto (2, -3) não está na curva dada. Ponha as coordenadas (2, -3) na equação dada: LHS = 2 (16) (4) (81) +6 (8) +7 (9) = 10368 +48 +63 = 10479 !! = 2703 Então o ponto (2, -3) não está na curva dada. Consulte Mais informação »

Como você diferencia implicitamente 9 = e ^ (y ^ 2-y) / e ^ x + y-xy?

Como você diferencia implicitamente 9 = e ^ (y ^ 2-y) / e ^ x + y-xy?

9 = e ^ (y ^ 2-y) / e ^ x + y - xy 9 = e ^ (y ^ 2-y) * e ^ (- x) + y - xy 9 = e ^ (y ^ 2 yx) + y - xy Diferencie em relação a x. A derivada do exponencial é ela mesma, vezes a derivada do expoente. Lembre-se que sempre que você diferencia algo que contém y, a regra da cadeia fornece um fator de y '. 0 = e ^ (y ^ 2-yx) (2yy '-y'-1) + y' - (xy '+ y) 0 = e ^ (y ^ 2-yx) (2yy' -y'-1) + y '- xy'-y Agora resolva para y'. Aqui está um começo: 0 = 2yy'e ^ (y ^ 2-yx) -y'e ^ (y ^ 2-yx) -e ^ (y ^ 2-yx) + y '- xy'-y Obter todos os termos tendo y Consulte Mais informação »

Diferencie a função. Y = x (x-4)?

Diferencie a função. Y = x (x-4)?

Comece usando a propriedade distributiva. Seja y = sqrtx (x - 4) Então y = xsqrtx - 4sqrtx = x ^ (3/2) - 4x ^ (1/2) Diferencie usando a regra de energia. dy / dx = (3/2) x ^ (1/2) - 2x ^ (- 1/2) = (3/2) x ^ (1/2) - 2 / x ^ (1/2) = ( 3sqrtx / 2) - 2 / sqrtx Obtenha um denominador comum de 2sqrtx e você chegará à sua resposta. Consulte Mais informação »

Como resolver para inte ^ xcosxdx?

Como resolver para inte ^ xcosxdx?

Int e ^ x cos (x) "d" x = 1 / 2e ^ x (sen (x) + cos (x)) + IC = int e ^ x cos (x) "d" x estar usando integração por partes, que afirma que int u "d" v = uv-int v "d" u. Use integração por partes, com u = e ^ x, du = e ^ x "d" x, "d" v = cos (x) "d" x, e v = sin (x): I = e ^ xsin (x) -int e ^ xsin (x) "d" x Use a integração por partes novamente para a segunda integral, com u = e ^ x, "d" u = e ^ x "d" x, " d "v = sen (x) " d "x, e v = -cos (x): I = e ^ xsin (x) + e ^ xcos (x) Consulte Mais informação »

Se quisermos aproximar o valor de cos 20 ° com um polinômio, qual o grau mínimo deve ser o polinômio para que o erro seja menor que 10 ^ -3?

Se quisermos aproximar o valor de cos 20 ° com um polinômio, qual o grau mínimo deve ser o polinômio para que o erro seja menor que 10 ^ -3?

0 "Esta questão é mal colocada como" 0,93969 "é um polinômio de grau 0 que faz o trabalho." "Uma calculadora calcula o valor de cos (x) através da série" "de Taylor." "A série de Taylor de cos (x) é:" 1 - x ^ 2 / (2!) + X ^ 4 / (4!) - x ^ 6 / (6!) + ... "O que você precisa saber é que o ângulo que você preenche nesta série "" deve ser em radianos. Então 20 ° = "pi / 9 = 0.349 ..." rad. " "Para ter uma série convergente rápida | x | deve ser menor que 1,&qu Consulte Mais informação »

Qual é a equação da linha tangente de f (x) = 6x-x ^ 2 em x = -1?

Qual é a equação da linha tangente de f (x) = 6x-x ^ 2 em x = -1?

Veja abaixo: O primeiro passo é encontrar a primeira derivada de f. f (x) = 6x-x ^ 2 f '(x) = 6-2x Assim: f' (- 1) = 6 + 2 = 8 O valor da significância de 8 é que este é o gradiente de f onde x = - 1 Este é também o gradiente da linha tangente que toca o gráfico de f nesse ponto. Portanto, nossa função de linha é atualmente y = 8x No entanto, também devemos encontrar a interceptação de y, mas para fazer isso, também precisamos da coordenada y do ponto onde x = -1. Conecte x = -1 em f. f (-1) = - 6- (1) = - 7 Então, um ponto na linha tangente Consulte Mais informação »

Qual é a inclinação da linha tangente de xy ^ 2- (1-xy) ^ 2 = C, onde C é uma constante arbitrária, em (1, -1)?

Qual é a inclinação da linha tangente de xy ^ 2- (1-xy) ^ 2 = C, onde C é uma constante arbitrária, em (1, -1)?

Dy / dx = -1,5 Primeiro encontramos d / dx de cada termo. d / dx [xy ^ 2] -d / dx [(1-xy) ^ 2] = d / dx [C] d / dx [x] y ^ 2 + d / dx [y ^ 2] x-2 ( 1-xy) d / dx [1-xy] = 0 y ^ 2 + d / dx [y ^ 2] x-2 (1-xy) (d / dx [1] -d / dx [xy]) = 0 y ^ 2 + d / dx [y ^ 2] x-2 (1-xy) (- d / dx [x] y + d / dx [y] x) = 0 y ^ 2 + d / dx [y ^ 2] x-2 (1-xy) (- y + d / dx [y] x) = 0 A regra da cadeia informa: d / dx = d / dy * dy / dx y ^ 2 + dy / dx d / dy [y ^ 2] x-2 (1-xy) (- y + dy / dxd / dy [y] x) = 0 y ^ 2 + dy / dx 2yx-2 (1-xy) (- y + dy / dx x) = 0 dy / dx 2yx-2 (1-x) dy / dx x = -y ^ 2-2y (1-xy) dy / dx (2yx-2x (1-x)) = - y ^ 2-2y (1 Consulte Mais informação »

A sequência a_n = (1 + 3 / n) ^ (4n) é convergente ou divergente?

A sequência a_n = (1 + 3 / n) ^ (4n) é convergente ou divergente?

"Ver explicação" a_n = ((1 + 3 / n) ^ 4) ^ n = (((1 + 3 / n) ^ 2) ^ 2) ^ n = ((1 + 6 / n + 9 / n ^ 2) ^ 2) ^ n = (1 + 36 / n ^ 2 + 81 / n ^ 4 + 12 / n + 18 / n ^ 2 + 108 / n ^ 3) ^ n = (1 + 12 / n + 54 / n ^ 2 + 108 / n ^ 3 + 81 / n ^ 4) ^ n "Note que você poderia mais facilmente aplicar o limite de Euler aqui:" lim_ {n-> oo} (1 + 1 / n) ^ n = e = 2,7182818 .... => lim_ {n-> oo} (1 + 3 / n) ^ (12 * n / 3) = e ^ 12 = 162754.79 .... "Então a sequência cresce muito, mas não infinitamente grande, "" converge. " Consulte Mais informação »

É a série sum_ (n = 0) ^ infty1 / ((2n + 1)!) Absolutamente convergente, condicionalmente convergente ou divergente?

É a série sum_ (n = 0) ^ infty1 / ((2n + 1)!) Absolutamente convergente, condicionalmente convergente ou divergente?

"Compare com" sum_ {n = 0} ^ oo 1 / (n!) = Exp (1) = e = 2,7182818 ... "Cada termo é igual ou menor que o" sum_ {n = 0} ^ oo 1 / (n!) = Exp (1) = e = 2.7182818 ... "Todos os termos são positivos, então a soma S da série é entre" 0 <S <e = 2.7182818 .... "Então a série é absolutamente convergente." Consulte Mais informação »

Quais são os pontos de inflexão, se houver, de f (x) = 2x ^ 4-e ^ (8x?

Quais são os pontos de inflexão, se houver, de f (x) = 2x ^ 4-e ^ (8x?

Veja abaixo O primeiro passo é encontrar a segunda derivada da função f (x) = 2x ^ 4-e ^ (8x) f '(x) = 8x ^ 3-8e ^ (8x) f' '(x) = 24x ^ 2-64e ^ (8x) Então devemos encontrar um valor de x onde: f '' (x) = 0 (usei uma calculadora para resolver isso) x = -0.3706965 Então, no valor x dado, a segunda derivada é 0. No entanto, para que seja um ponto de inflexão, deve haver uma mudança de sinal em torno desse valor x. Por isso podemos ligar valores na função e ver o que acontece: f (-1) = 24-64e ^ (- 8) definitivamente positivo como 64e ^ (- 8) é muito peq Consulte Mais informação »

Como você encontra o volume do sólido obtido girando a região limitada por y = x e y = x ^ 2 sobre o eixo x?

Como você encontra o volume do sólido obtido girando a região limitada por y = x e y = x ^ 2 sobre o eixo x?

V = (2pi) / 15 Primeiro precisamos dos pontos onde x e x ^ 2 se encontram. x = x ^ 2 x ^ xx = 0 x (x-1) = 0 x = 0 ou 1 Portanto, nossos limites são 0 e 1. Quando temos duas funções para o volume, usamos: V = piint_a ^ b (f (x) ^ 2-g (x) ^ 2) dx V = piint_0 ^ 1 (x ^ 2-x ^ 4) dx V = pi [x ^ 3/3-x ^ 5/5] _0 ^ 1 V = pi (1 / 3-1 / 5) = (2pi) / 15 Consulte Mais informação »

Como você diferencia y = (x + 5) (2x-3) (3x ^ 2 + 4)?

Como você diferencia y = (x + 5) (2x-3) (3x ^ 2 + 4)?

Y '= (2x-3) (3x ^ 2 + 4) +2 (x + 5) (3x ^ 2 + 4) + 6x (2x-3) (x + 5) y' = 24x ^ 3 + 63x ^ 2-74x + 28 Se y = uvw, onde u, v, e w são todas funções de x, então: y '= uvw' + uv'w + u'vw (Isso pode ser encontrado fazendo uma regra de cadeia com dois funções substituídas como uma, ie fazendo uv = z) u = x + 5 u '= 1 v = 2x-3 v' = 2 w = 3x ^ 2 + 4 w '= 6x y' = (2x-3) (3x ^ 2 + 4) +2 (x + 5) (3x ^ 2 + 4) + 6x (2x-3) (x + 5) y '= 6x ^ 3 + 8x-9x ^ 2-12 + 6x ^ 3 + 8x + 30x ^ 2 + 40 + 12x ^ 3 + 60x ^ 2-18x ^ 2-90x y '= 24x ^ 3 + 63x ^ 2-74x + 28 Consulte Mais informação »

Como você diferencia implicitamente 2x / y = ysqrt (x ^ 2 + y ^ 2) -x?

Como você diferencia implicitamente 2x / y = ysqrt (x ^ 2 + y ^ 2) -x?

Dy / dx = - (yx (x ^ 2 + y ^ 2) ^ (- 1/2) -1-2y ^ -1) / (xy ^ -2- (x ^ 2 + y ^ 2) ^ (1 / 2) + y ^ 2 (x ^ 2 + y ^ 2) ^ (- 1/2)) Tudo bem, isso é muito longo. Vou numerar cada passo para facilitar, e também não combinei os passos para que você soubesse o que estava acontecendo. Comece com: 2xy ^ -1 = y (x ^ 2 + y ^ 2) ^ (1/2) -x Primeiro tomamos d / dx de cada termo: 2. d / dx [2xy ^ -1] = d / dx [y (x ^ 2 + y ^ 2) ^ (1/2)] - d / dx [x] 3. d / dx [2x] y ^ -1 + xd / dx [y ^ -1] = d / dx [y] (x ^ 2 + y ^ 2) ^ (1/2) + yd / dx [(x ^ 2 + y ^ 2) ^ (1/2)] - d / dx [x] 4. 2y ^ -1 + xd / dx [y ^ -1] = d / dx [y] ( Consulte Mais informação »

Qual é a equação da linha tangente de f (x) = sqrt (x ^ 2e ^ x) em x = 3?

Qual é a equação da linha tangente de f (x) = sqrt (x ^ 2e ^ x) em x = 3?

Y = 11.2x-20.2 Ou y = (5e ^ (3/2)) / 2x-2e ^ (3/2) y = e ^ (3/2) ((5x) / 2-2) Temos: f (x) = (x ^ 2e ^ x) ^ (1/2) f '(x) = (x ^ 2e ^ x) ^ (- 1/2) / 2 * d / dx [x ^ 2e ^ x] f '(x) = (x ^ 2e ^ x) ^ (- 1/2) / 2 * (2xe ^ x + x ^ 2e ^ x) f' (x) = ((2xe ^ x + x ^ 2e ^ x) (x ^ 2e ^ x) ^ (- 1/2)) / 2 f '(x) = (2xe ^ x + x ^ 2e ^ x) / (2 (x ^ 2e ^ x) ^ (1 / 2)) = (2xe ^ x + x ^ 2e ^ x) / (2sqrt (x ^ 2e ^ x)) f '(3) = (2 (3) e ^ 3 + 3 ^ 2e ^ 3) / (2sqrt (3 ^ 2e ^ 3)) = (5e ^ (3/2)) / 2 ~ ~ 11,2 y = mx + cf (3) = sqrt (9e ^ 3) = 3e ^ (3/2) ~ ~ 13,4 13,4 = 11,2 (3) + cc = 13,4-11,2 (3) = - 20,2 y = 11,2x-20,2 Ou y Consulte Mais informação »

Como você diferencia f (x) = (5e ^ x + tanx) (x ^ 2-2x) usando a regra do produto?

Como você diferencia f (x) = (5e ^ x + tanx) (x ^ 2-2x) usando a regra do produto?

F '(x) = (5e ^ x + seg ^ 2x) (x ^ 2-2x) + (5e ^ x + tanx) (2x-2) Para f (x) = (5e ^ x + tanx) (x ^ 2-2x), encontramos f '(x) fazendo: f' (x) = d / dx [5e ^ x + tanx] (x ^ 2-2x) + (5e ^ x + tanx) d / dx [x ^ 2-2x] f '(x) = (5e ^ x + seg ^ 2x) (x ^ 2-2x) + (5e ^ x + tanx) (2x-2) Consulte Mais informação »

Qual é a série de Taylor de f (x) = arctan (x)?

Qual é a série de Taylor de f (x) = arctan (x)?

F (x) = sum_ {n = 1} ^ infty (-1) ^ n {x ^ {2n + 1}} / {2n + 1} Vamos ver alguns detalhes. f (x) = arctanx f '(x) = 1 / {1 + x ^ 2} = 1 / {1 - (- x ^ 2)} Lembre-se que a série de potência geométrica 1 / {1-x} = sum_ { n = 0} ^ infty x ^ n substituindo x por -x ^ 2, Rightarrow 1 / {1 - (- x ^ 2)} = sum_ {n = 0} ^ infty (-x ^ 2) ^ n = sum_ {n = 0} ^ infty (-1) ^ nx ^ {2n} Então, f '(x) = sum_ {n = 0} ^ infty (-1) ^ nx ^ {2n} Ao integrar f (x) = int sum_ {n = 0} ^ infty (-1) ^ nx ^ {2n} dx colocando o sinal integral dentro do somatório, = sum_ {n = 0} ^ infty int (-1) ^ nx ^ {2n} dx por Power Consulte Mais informação »

Qual é o valor de? lim_ (x-> 0) (int_0 ^ x sen t ^ 2.dt) / sin x ^ 2

Qual é o valor de? lim_ (x-> 0) (int_0 ^ x sen t ^ 2.dt) / sin x ^ 2

Lim_ (x rarr 0) (int_0 ^ x sen t ^ 2 dt) / (sen x ^ 2) = 0 Procuramos: L = lim_ (x rarr 0) (int_0 ^ x sen t ^ 2 dt) / (sen x ^ 2) Tanto o numerador quanto o denominador2 rarr 0 como x rarr 0. Assim, o limite L (se existir) é de uma forma indeterminada 0/0 e, consequentemente, podemos aplicar a regra de L'Hôpital para obter: L = lim_ (x rarr 0) (d / dx int_0 ^ x sen (t ^ 2) dt) / (d / dx sen (x ^ 2)) = lim_ (x rarr 0) (d / dx int_0 ^ x sen ( t ^ 2) dt) / (d / dx sen (x ^ 2)) Agora, usando o teorema fundamental do cálculo: d / dx int_0 ^ x sen (t ^ 2) dt = sen (x ^ 2) E, d / dx sin (x ^ 2) = 2xcos (x ^ 2) Consulte Mais informação »

Qual é o valor de F '(x) se F (x) = int_0 ^ sinxsqrt (t) dt?

Qual é o valor de F '(x) se F (x) = int_0 ^ sinxsqrt (t) dt?

: F '(x) = (sqrtsinx) (cosx). F (x) = int_0 ^ sinx sqrttdt porque, intsqrttdt = intt ^ (1/2) dt = t ^ (1/2 + 1) / (1/2 + 1) = 2 / 3t ^ (3/2) + c,:. F (x) = [2 / 3t ^ (3/2)] _ 0 ^ sinx:. F (x) = 2 / 3sin ^ (3/2) x:. F '(x) = 2/3 [{(sinx)} ^ (3/2)]' Usando a Regra da Corrente, F '(x) = 2/3 [3/2 (sinx) ^ (3 / 2- 1)] d / dx (senx) = (senx) ^ (1/2) (cosx):. F '(x) = (sqrtsinx) (cosx). Desfrute de matemática! Consulte Mais informação »

Como você encontra o limite lim_ (h-> 0) ((2 + h) ^ 3-8) / h?

Como você encontra o limite lim_ (h-> 0) ((2 + h) ^ 3-8) / h?

12 Podemos expandir o cubo: (2 + h) ^ 3 = 8 + 12h + 6h ^ 2 + h ^ 3 Conectando isto, lim_ (hrightarrow 0) (8 + 12h + 6h ^ 2 + h ^ 3-8) / h = lim_ (hrightarrow 0) (12h + 6h ^ 2 + h ^ 3) / h = lim_ (hrightarrow 0) (12 + 6h + h ^ 2) = 12. Consulte Mais informação »

Como você encontra o limite lim_ (h-> 0) (sqrt (1 + h) -1) / h?

Como você encontra o limite lim_ (h-> 0) (sqrt (1 + h) -1) / h?

Frac {1} {2} O limite apresenta uma forma indefinida 0/0. Neste caso, você pode usar o teorema de l'hospital, que indica lim frac {f (x)} {g (x)} = lim frac {f '(x)} {g' (x)} derivativo do numerador é frac {1} {2sqrt (1 + h)} Enquanto a derivada do denominador é simplesmente 1. Então, lim_ {x para 0} frac {f '(x)} {g' (x)} = lim_ {x para 0} frac { frac {1} {2sqrt (1 + h)}} {1} = lim_ {x para 0} frac {1} {2sqrt ( 1 + h)} E assim simplesmente frac {1} {2sqrt (1)} = frac {1} {2} Consulte Mais informação »

Como você encontra o limite lim_ (x-> 2) (x ^ 2 + x-6) / (x-2)?

Como você encontra o limite lim_ (x-> 2) (x ^ 2 + x-6) / (x-2)?

Comece por fatorar o numerador: = lim_ (x-> 2) (((x + 3) (x-2)) / (x-2)) Podemos ver que o termo (x - 2) será cancelado. Portanto, este limite é equivalente a: = lim_ (x-> 2) (x + 3) Agora deve ser fácil ver o que o limite avalia: = 5 Vamos dar uma olhada em um gráfico de como esta função seria , para ver se nossa resposta concorda: O "buraco" em x = 2 é devido ao termo (x - 2) no denominador. Quando x = 2, este termo se torna 0, e uma divisão por zero ocorre, resultando na função sendo indefinida em x = 2. Entretanto, a função é bem definida Consulte Mais informação »

Como você encontra o limite lim_ (x -> - 4) (x ^ 2 + 5x + 4) / (x ^ 2 + 3x-4)?

Como você encontra o limite lim_ (x -> - 4) (x ^ 2 + 5x + 4) / (x ^ 2 + 3x-4)?

= 3/5 Explicação, Usando Finding Limits Algebricamente, = lim_ (x -> - 4) (x ^ 2 + 5x + 4) / (x ^ 2 + 3x-4), se ligarmos x = -4, obtemos 0/0 form = lim_ (x -> - 4) (x ^ 2 + 4x + x + 4) / (x ^ 2 + 4x-x-4) = lim_ (x -> - 4) (x (x + 4) +1 (x + 4)) / (x (x + 4) -1 (x + 4)) = lim_ (x -> - 4) ((x + 4) (x + 1)) / (( x + 4) (x-1)) = lim_ (x -> - 4) ((x + 1)) / ((x-1)) = (- 3) / - 5 = 3/5 Consulte Mais informação »

Como você encontra o limite lim_ (x-> 4) (x ^ 3-64) / (x ^ 2-8x + 16)?

Como você encontra o limite lim_ (x-> 4) (x ^ 3-64) / (x ^ 2-8x + 16)?

Primeiro fator o denominador ... (x ^ 3-64) / ((x-4) (x-4)) Agora fator o numerador ... ((x-4) (x ^ 2 + 4x + 16)) / ((x-4) (x-4)) Divida o numerador e o denominador por x-4 ... (x ^ 2 + 4x + 16) / (x-4) Substitua todos os xs pelo limite que está sendo aproximado (4) ... ((4) ^ 2 + 4 (4) +16) / ((4) -4) Combina termos ... 48/0 O limite se aproxima do infinito desde que a divisão por 0 é indefinida, mas a divisão por 0 também se aproxima infinidade. Consulte Mais informação »

É f (x) = - 4x ^ 3 + 4x ^ 2 + 2x-1 aumentando ou diminuindo em x = 2?

É f (x) = - 4x ^ 3 + 4x ^ 2 + 2x-1 aumentando ou diminuindo em x = 2?

Está diminuindo. Comece por derivar a função f, como função derivada, f 'descreve a taxa de mudança de f. f (x) = - 4x ^ 3 + 4x ^ 2 + 2x-1 f '(x) = - 12x ^ 2 + 8x + 2 Em seguida, insira x = 2 na função. f '(2) = - 12 (4) +8 (2) +2 f' (2) = - 48 + 18 f´ (2) = - 30 Portanto, como o valor da derivada é negativo, a taxa instantânea de mudança neste ponto é negativo - então a função de f está diminuindo nesta instância. Consulte Mais informação »

Qual é a derivada da função f (x) = ln (ln ((x + 4) / ln (x ^ 2 + 4)?

Qual é a derivada da função f (x) = ln (ln ((x + 4) / ln (x ^ 2 + 4)?

F '(x) = (1 / (ln ((x + 4) / (ln (x ^ 2 + 4))))) ((1) / ((x + 4))). (((x ^ 2 + 4) (ln (x ^ 2 + 4)) - (2x ^ 2 + 4x)) / ((x ^ 2 + 4) (ln (x ^ 2 + 4)))) f '(x) = (1 / (ln ((x + 4) / (ln (x ^ 2 + 4))))) (1 / ((x + 4) / (ln (x ^ 2 + 4)))). (1) (ln (x ^ 2 + 4)) - (x + 4) (1) / ((x ^ 2 + 4)) (2x)) / ((ln (x ^ 2 + 4))) 2) f '(x) = (1 / (ln ((x + 4) / (ln (x ^ 2 + 4))))) (ln (x ^ 2 + 4) / ((x + 4)) ) ((ln (x ^ 2 + 4) - (2x ^ 2 + 4x) / ((x ^ 2 + 4))) / ((ln (x ^ 2 + 4))) ^ 2) f '( x) = (1 / (ln ((x + 4) / (ln (x ^ 2 + 4))))) (cancelar (ln (x ^ 2 + 4)) / ((x + 4))). (((x ^ 2 + 4) (ln (x ^ 2 + 4)) - (2x ^ 2 + 4x)) / (( Consulte Mais informação »

Como você testa a convergência para 1 / ((2n + 1)!)?

Como você testa a convergência para 1 / ((2n + 1)!)?

No caso você quis dizer "testar a convergência da série: sum_ (n = 1) ^ (oo) 1 / ((2n + 1)!)" A resposta é: cor (azul) "converge" Para descobrir, podemos usar o teste de razão.Isto é, se "U" _ "n" é o termo n ^ "th" desta série Então se, mostramos que lim_ (nrarr + oo) abs ("U" _ ("n" +1) / "U "_n) <1 significa que a série converge No outro se lim_ (nrarr + oo) abs ((" U "_ (" n "+1)) /" U "_n)> 1 significa que a série diverge No nosso caso "U" Consulte Mais informação »

Int2 / (2x ^ 2 + 2x) dx?

Int2 / (2x ^ 2 + 2x) dx?

Ln (abs (x / (x + 1))) + C Primeiro fatoramos 2: int1 / (x ^ 2 + x) dx Então fatorizemos o denominador: int1 / (x (x + 1)) dx Precisamos dividir isso em frações parciais: 1 = A (x + 1) + Bx Usando x = 0 nos dá: A = 1 Então usando x = -1 nos dá: 1 = -B Usando isto nós obtemos: int1 / x-1 / (x + 1) dx int1 / xdx-int / (x + 1) dx ln (abs (x)) - ln (abs (x + 1) + C ln (abs (x / (x + 1))) + C Consulte Mais informação »

O que é uma Asymptote Vertical?

O que é uma Asymptote Vertical?

Uma assíntota vertical é uma linha vertical que ocorre em x = c, onde c é um número real, se o limite da função f (x) se aproxima de + -oo como x-> c da esquerda ou da direita (ou de ambos) . Para uma explicação mais completa das assíntotas verticais, acesse: http://socratic.org/questions/what-is-a-vertical-asymptote-in-calculus? Consulte Mais informação »